PT 47 LR 1 Q #14 Forum

Prepare for the LSAT or discuss it with others in this forum.
Post Reply
lawschoolisfun2012

New
Posts: 99
Joined: Sun Aug 23, 2009 2:56 am

PT 47 LR 1 Q #14

Post by lawschoolisfun2012 » Sun Aug 01, 2010 2:12 pm

Assumption (Type 2) question. I could not pick between (A) and (E). Both answer choices seem to be necessary assumptions for the argument. Perhaps understanding the difference between the two would help. Thanks in advance. PT 47 LR 1 Q #14, correct answer and why?

lawschoolisfun2012

New
Posts: 99
Joined: Sun Aug 23, 2009 2:56 am

Re: PT 47 LR 1 Q #14

Post by lawschoolisfun2012 » Sun Aug 01, 2010 11:40 pm

If you teach someone how to answer a question, you are more likely to retain and better understand the information yourself...So, someone please help you and me? :)

sjlawand88

New
Posts: 36
Joined: Fri Jun 25, 2010 4:33 pm

Re: PT 47 LR 1 Q #14

Post by sjlawand88 » Sun Aug 01, 2010 11:57 pm

Correct answer is E. A was also a contender for me. Look closely at the argument's first sentence and last sentence. It is NOT assumed that local population MUST have different local population dialects. Last sentence states: "It will develop many regional dialects." A is almost explicitly stated in the argument.

Post Reply

Return to “LSAT Prep and Discussion Forum”